Exam 3 MedSurg

Réussis tes devoirs et examens dès maintenant avec Quizwiz!

A preceptor is discussing stroke with a new nurse on the unit. The preceptor would tell the new nurse which cardiac dysrhythmia is associated with cardiogenic embolic strokes? A) Ventricular tachycardia B) Atrial fibrillation C) Supraventricular tachycardia D) Bundle branch block

B) Atrial fibrillation

To alleviate pain associated with trigeminal neuralgia, a patient is taking Tegretol (carbamazepine). What health education should the nurse provide to the patient before initiating this treatment? A) Concurrent use of calcium supplements is contraindicated. B) Blood levels of the drug must be monitored. C) The drug is likely to cause hyperactivity and agitation. D) Tegretol can cause tinnitus during the first few days of treatment.

B) Blood levels of the drug must be monitored.

A patient diagnosed with MS has been admitted to the medical unit for treatment of an MS exacerbation. Included in the admission orders is baclofen (Lioresal). What should the nurse identify as an expected outcome of this treatment? A) Reduction in the appearance of new lesions on the MRI B) Decreased muscle spasms in the lower extremities C) Increased muscle strength in the upper extremities D) Decreased severity and duration of exacerbations

B) Decreased muscle spasms in the lower extremities

The nurse is planning the care of a patient who has been recently diagnosed with a cerebellar tumor. Due to the location of this patient's tumor, the nurse should implement measures to prevent what complication? A) Falls B) Audio hallucinations C) Respiratory depression D) Labile BP

A) Falls

A patient has recently begun mobilizing during the recovery from an ischemic stroke. To protect the patient's safety during mobilization, the nurse should perform what action? A) Support the patient's full body weight with a waist belt during ambulation. B) Have a colleague follow the patient closely with a wheelchair. C) Avoid mobilizing the patient in the early morning or late evening. D) Ensure that the patient's family members do not participate in mobilization.

B) Have a colleague follow the patient closely with a wheelchair.

The nurse is caring for a patient who is scheduled for a cervical discectomy the following day. During health education, the patient should be made aware of what potential complications? A) Vertebral fracture B) Hematoma at the surgical site C) Scoliosis D) Renal trauma

B) Hematoma at the surgical site

The nurse in an extended care facility is planning the daily activities of a patient with postpolio syndrome. The nurse recognizes the patient will best benefit from physical therapy when it is scheduled at what time? A) Immediately after meals B) In the morning C) Before bedtime D) In the early evening

B) In the morning

The nurse is providing care for a patient who is unconscious. What nursing intervention takes highest priority? A) Maintaining accurate records of intake and output B) Maintaining a patent airway C) Inserting a nasogastric (NG) tube as ordered D) Providing appropriate pain control

B) Maintaining a patent airway

A male patient presents to the clinic complaining of a headache. The nurse notes that the patient is guarding his neck and tells the nurse that he has stiffness in the neck area. The nurse suspects the patient may have meningitis. What is another well-recognized sign of this infection? A) Negative Brudzinski's sign B) Positive Kernig's sign C) Hyperpatellar reflex D) Sluggish pupil reaction

B) Positive Kernig's sign

The nurse is caring for a patient diagnosed with Parkinson's disease. The patient is having increasing problems with rising from the sitting to the standing position. What should the nurse suggest to the patient to use that will aid in getting from the sitting to the standing position as well as aid in improving bowel elimination? A) Use of a bedpan B) Use of a raised toilet seat C) Sitting quietly on the toilet every 2 hours D) Following the outlined bowel program

B) Use of a raised toilet seat

A patient with a documented history of seizure disorder experiences a generalized seizure. What nursing action is most appropriate? A) Restrain the patient to prevent injury. B) Open the patient's jaws to insert an oral airway. C) Place patient in high Fowler's position. D) Loosen the patient's restrictive clothing.

D) Loosen the patient's restrictive clothing.

A patient, diagnosed with cancer of the lung, has just been told he has metastases to the brain. What change in health status would the nurse attribute to the patient's metastatic brain disease? A) Chronic pain B) Respiratory distress C) Fixed pupils D) Personality changes

D) Personality changes

A patient suffers a head injury. The nurse implements an assessment plan to monitor for potential subdural hematoma development. Which manifestation does the nurse anticipate seeing first?

- Change in LOC

15. A client with a spinal cord injury is to receive Lovenox (enoxaparin) 50 mg subcutaneously twice a day. The medication is supplied in vials containing 80 mg per 0.8 mL. How many mL will constitute the correct dose? Enter the correct number ONLY.

0.5

You are the clinic nurse caring for a patient with a recent diagnosis of myasthenia gravis. The patient has begun treatment with pyridostigmine bromide (Mestinon). What change in status would most clearly suggest a therapeutic benefit of this medication? A) Increased muscle strength B) Decreased pain C) Improved GI function D) Improved cognition

A) Increased muscle strength

A patient with diabetes presents to the clinic and is diagnosed with a mononeuropathy. This patient's nursing care should involve which of the following? A) Protection of the affected limb from injury B) Passive and active ROM exercises for the affected limb C) Education about improvements to glycemic control D) Interventions to prevent contractures

A) Protection of the affected limb from injury

The child is having a seizure activity displayed as suddenly dropping a cup of soda but not falling. Her face looked blank and the eyelids were twitching. The child does not recall anything after the seizure denies an aura and was not incontinent. What term would best describe this seizure?

Absence Seizure

The nurse witnesses a patient with a seizure disorder as the patient suddenly jerks the arms and legs, falls to the floor, and then regains consciousness immediately. The first initial action by the nurse should be:

Assess the patient for a possible head injury

Which one of the following signs and symptoms would be an indication of increased intracranial pressure in infants?

Bulging fontanels

A patient diagnosed with transient ischemic attacks (TIAs) is scheduled for a carotid endarterectomy. The nurse explains that this procedure will be done for what purpose? A) To decrease cerebral edema B) To prevent seizure activity that is common following a TIA C) To remove atherosclerotic plaques blocking cerebral flow D) To determine the cause of the TIA

C) To remove atherosclerotic plaques blocking cerebral flow

The public health nurse is planning a health promotion campaign that reflects current epidemiologic trends. The nurse should know that hemorrhagic stroke currently accounts for what percentage of total strokes in the United States? A) 43% B) 33% C) 23% D) 13%

D) 13%

The nurse is preparing to provide care for a patient diagnosed with myasthenia gravis. The nurse should know that the signs and symptoms of the disease are the result of what? A) Genetic dysfunction B) Upper and lower motor neuron lesions C) Decreased conduction of impulses in an upper motor neuron lesion D) A lower motor neuron lesion

D) A lower motor neuron lesion

A patient diagnosed with a cerebral aneurysm reports a severe headache to the nurse. What action is a priority for the nurse? A) Sit with the patient for a few minutes. B) Administer an analgesic. C) Inform the nurse-manager. D) Call the physician immediately.

D) Call the physician immediately.

The nurse notes in the patient's medical history that the patient has a positive Romberg test. Which diagnosis is appropriate?

Risk for falls related to dizziness or weakness

16. Autonomic dysreflexia can occur with spinal cord injuries above which of the following levels?

T6

3. Which of the following methods may be used by the nurse to maintain the peripheral circulation in a patient with increased intracerebral pressure (ICP)?

a. Apply elastic stockings to lower extremities

8. Which of the following is the most common cause of spinal cord injury (SCI)?

a. Motor vehicle crashes

A patient with a new diagnosis of amyotrophic lateral sclerosis (ALS) is overwhelmed by his diagnosis and the known complications of the disease. How can the patient best make known his wishes for care as his disease progresses? A) Prepare an advance directive. B) Designate a most responsible physician (MRP) early in the course of the disease. C) Collaborate with representatives from the Amyotrophic Lateral Sclerosis Association. D) Ensure that witnesses are present when he provides instruction.

A) Prepare an advance directive.

A patient who suffered an ischemic stroke now has disturbed sensory perception. What principle should guide the nurse's care of this patient? A) The patient should be approached on the side where visual perception is intact. B) Attention to the affected side should be minimized in order to decrease anxiety. C) The patient should avoid turning in the direction of the defective visual field to minimize shoulder subluxation. D) The patient should be approached on the opposite side of where the visual perception is intact to promote recovery.

A) The patient should be approached on the side where visual perception is intact.

The nurse is developing a plan of care for a patient with Guillain-Barré syndrome. Which of the following interventions should the nurse prioritize for this patient? A) Using the incentive spirometer as prescribed B) Maintaining the patient on bed rest C) Providing aids to compensate for loss of vision D) Assessing frequently for loss of cognitive function

A) Using the incentive spirometer as prescribed

A patient has developed diabetes insipidus after having increased ICP following head trauma. What nursing assessment best addresses this complication? A) Vigilant monitoring of fluid balance B) Continuous BP monitoring C) Serial arterial blood gases (ABGs) D) Monitoring of the patient's airway for patency

A) Vigilant monitoring of fluid balance

The nurse is planning discharge education for a patient with trigeminal neuralgia. The nurse knows to include information about factors that precipitate an attack. What would the nurse be correct in teaching the patient to avoid? A) Washing his face B) Exposing his skin to sunlight C) Using artificial tears D) Drinking large amounts of fluids

A) Washing his face

A patient with suspected Parkinson's disease is initially being assessed by the nurse. When is the best time to assess for the presence of a tremor? A) When the patient is resting B) When the patient is ambulating C) When the patient is preparing his or her meal tray to eat D) When the patient is participating in occupational therapy

A) When the patient is resting

A patient is brought to the ER following a motor vehicle accident in which he sustained head trauma. Preliminary assessment reveals a vision deficit in the patient's left eye. The nurse should associate this abnormal finding with trauma to which of the following cerebral lobes? A)Temporal B)Occipital C)Parietal D)Frontal

B The posterior lobe of the cerebral hemisphere is responsible for visual interpretation. The temporal lobe contains the auditory receptive areas. The parietal lobe contains the primary sensory cortex, and is essential to an individual's awareness of the body in space, as well as orientation in space and spatial relations. The frontal lobe functions in concentration, abstract thought, information storage or memory, and motor function.

While completing a health history on a patient who has recently experienced a seizure, the nurse would assess for what characteristic associated with the postictal state? A) Epileptic cry B) Confusion C) Urinary incontinence D) Body rigidity

B) Confusion

As a member of the stroke team, the nurse knows that thrombolytic therapy carries the potential for benefit and for harm. The nurse should be cognizant of what contraindications for thrombolytic therapy? Select all that apply. A) INR above 1.0 B) Recent intracranial pathology C) Sudden symptom onset D) Current anticoagulation therapy E) Symptom onset greater than 3 hours prior to admission

B) Recent intracranial pathology D) Current anticoagulation therapy E) Symptom onset greater than 3 hours prior to admission

The nurse is caring for a patient with multiple sclerosis (MS). The patient tells the nurse the hardest thing to deal with is the fatigue. When teaching the patient how to reduce fatigue, what action should the nurse suggest? A) Taking a hot bath at least once daily B) Resting in an air-conditioned room whenever possible C) Increasing the dose of muscle relaxants D) Avoiding naps during the day

B) Resting in an air-conditioned room whenever possible

A patient newly diagnosed with a cervical disk herniation is receiving health education from the clinic nurse. What conservative management measures should the nurse teach the patient to implement? A) Perform active ROM exercises three times daily. B) Sleep on a firm mattress. C) Apply cool compresses to the back of the neck daily. D) Wear the cervical collar for at least 2 hours at a time.

B) Sleep on a firm mattress.

A nurse is assessing reflexes in a patient with hyperactive reflexes. When the patient's foot is abruptly dorsiflexed, it continues to "beat" two to three times before settling into a resting position. How would the nurse document this finding? A) Rigidity B) Flaccidity C) Clonus D) Ataxia

C When reflexes are very hyperactive, a phenomenon called clonus may be elicited. If the foot is abruptly dorsiflexed, it may continue to "beat" two to three times before it settles into a position of rest. Rigidity is an increase in muscle tone at rest characterized by increased resistance to passive stretch. Flaccidity is lack of muscle tone. Ataxia is the inability to coordinate muscle movements, resulting in difficulty walking, talking, and performing self-care activities.

The nurse is discharging a patient home after surgery for trigeminal neuralgia. What advice should the nurse provide to this patient in order to reduce the risk of injury? A) Avoid watching television or using a computer for more than 1 hour at a time. B) Use OTC antibiotic eye drops for at least 14 days. C) Avoid rubbing the eye on the affected side of the face. D) Rinse the eye on the affected side with normal saline daily for 1 week.

C) Avoid rubbing the eye on the affected side of the face.

The nurse is caring for a patient with a brain tumor. What drug would the nurse expect to be ordered to reduce the edema surrounding the tumor? A) Solumedrol B) Dextromethorphan C) Dexamethasone D) Furosemide

C) Dexamethasone

The nurse is preparing health education for a patient who is being discharged after hospitalization for a hemorrhagic stroke. What content should the nurse include in this education? A) Mild, intermittent seizures can be expected. B) Take ibuprofen for complaints of a serious headache. C) Take antihypertensive medication as ordered. D) Drowsiness is normal for the first week after discharge.

C) Take antihypertensive medication as ordered.

The nurse is writing a care plan for a patient with brain metastases. The nurse decides that an appropriate nursing diagnosis is "anxiety related to lack of control over the health circumstances." In establishing this plan of care for the patient, the nurse should include what intervention? A) The patient will receive antianxiety medications every 4 hours. B) The patient's family will be instructed on planning the patient's care. C) The patient will be encouraged to verbalize concerns related to the disease and its treatment. D) The patient will begin intensive therapy with the goal of distraction.

C) The patient will be encouraged to verbalize concerns related to the disease and its treatment.

A patient who has been experiencing numerous episodes of unexplained headaches and vomiting has subsequently been referred for testing to rule out a brain tumor. What characteristic of the patient's vomiting is most consistent with a brain tumor? A) The patient's vomiting is accompanied by epistaxis. B) The patient's vomiting does not relieve his nausea. C) The patient's vomiting is unrelated to food intake. D) The patient's emesis is blood-tinged.

C) The patient's vomiting is unrelated to food intake.

23. Which type of brain injury has occurred if the patient may be aroused with effort, but soon slips back into unconsciousness?

Contusion i. A patient with a contusion may be aroused with effort but soon slips back into unconsciousness. A concussion is a temporary loss of neurologic function with no apparent structural damage. A diffuse axonal injury involves widespread damage to the axons in the cerebral hemispheres, corpus callosum, and brain stem. An intracranial hemorrhage is a collection of blood that develops within the cranial vault.

A patient with a cerebral aneurysm exhibits signs and symptoms of an increase in intracranial pressure (ICP). What nursing intervention would be most appropriate for this patient? A) Range-of-motion exercises to prevent contractures B) Encouraging independence with ADLs to promote recovery C) Early initiation of physical therapy D) Absolute bed rest in a quiet, nonstimulating environment

D) Absolute bed rest in a quiet, nonstimulating environment

The nurse is caring for a patient with Huntington disease who has been admitted to the hospital for treatment of malnutrition. What independent nursing action should be implemented in the patient's plan of care? A) Firmly redirect the patient's head when feeding. B) Administer phenothiazines after each meal as ordered. C) Encourage the patient to keep his or her feeding area clean. D) Apply deep, gentle pressure around the patient's mouth to aid swallowing.

D) Apply deep, gentle pressure around the patient's mouth to aid swallowing.

The nurse caring for a patient in ICU diagnosed with Guillain-Barré syndrome should prioritize monitoring for what potential complication? A) Impaired skin integrity B) Cognitive deficits C) Hemorrhage D) Autonomic dysfunction

D) Autonomic dysfunction

While assessing the patient at the beginning of the shift, the nurse inspects a surgical dressing covering the operative site after the patients' cervical diskectomy. The nurse notes that the drainage is 75% saturated with serosanguineous discharge. What is the nurse's most appropriate action? A) Page the physician and report this sign of infection. B) Reinforce the dressing and reassess in 1 to 2 hours. C) Reposition the patient to prevent further hemorrhage. D) Inform the surgeon of the possibility of a dural leak.

D) Inform the surgeon of the possibility of a dural leak.

A patient has just been diagnosed with Parkinson's disease and the nurse is planning the patient's subsequent care for the home setting. What nursing diagnosis should the nurse address when educating the patient's family? A) Risk for infection B) Impaired spontaneous ventilation C) Unilateral neglect D) Risk for injury

D) Risk for injury

The nurse caring for a patient diagnosed with Guillain-Barré syndrome is planning care with regard to the clinical manifestations associated this syndrome. The nurse's communication with the patient should reflect the possibility of what sign or symptom of the disease? A) Intermittent hearing loss B) Tinnitus C) Tongue enlargement D) Vocal paralysis

D) Vocal paralysis

A patient diagnosed with Bell's palsy is having decreased sensitivity to touch of the involved nerve. What should the nurse recommend to prevent atrophy of the muscles? A) Blowing up balloons B) Deliberately frowning C) Smiling repeatedly D) Whistling

D) Whistling

21. A client with quadriplegia is in spinal shock. What finding should the nurse expect?

a. Absence of reflexes along with flaccid extremities i. During the period immediately following a spinal cord injury, spinal shock occurs. In spinal shock, all reflexes are absent and the extremities are flaccid. When spinal shock subsides, the client will demonstrate positive Babinski's reflex, hyperreflexia, and spasticity of all four extremities.

19. A 65-year-old client was hit in the head with a ball and knocked unconscious. Upon her arrival at the emergency department and subsequent diagnostic tests, it was determined that she suffered a subdural hematoma. The client is becoming increasingly symptomatic. How would you expect her subdural hematoma to be classified?

a. Acute i. Subdural hematomas are classified as acute, subacute, and chronic according to the rate of neurologic changes. Symptoms progressively worsen in a client with an acute subdural hematoma within the first 24 hours of the head injury.

29. When the nurse observes that the patient has extension and external rotation of the arms and wrists, and extension, plantar flexion, and internal rotation of the feet, she records the patient's posturing as which of the following?

a. Decerebrate i. Decerebrate posturing is the result of lesions at the midbrain and is more ominous than decorticate posturing. The described posturing results from cerebral trauma and is not normal. The patient has no motor function, is limp, and lacks motor tone with flaccid posturing. In decorticate posturing, the patient has flexion and internal rotation of the arms and wrists and extension, internal rotation, and plantar flexion of the feet.

4. The nurse working on a neurological unit is mentoring a nursing student. The student asks about a client who has sustained a primary and and secondary brain injury. The nurse correctly tells the student which of the following, related to the primary injury?

a. It results from initial damage to the brain from the traumatic event

12. Which of the following is not a manifestation of Cushing's Triad?

a. Tachycardia i. Cushing's triad is manifested by bradycardia, hypertension, and bradypnea. Tachycardia is not a component of the triad

5. A patient has been diagnosed with a concussion. He is to be released from the emergency department. The nurse teaches the family or friends who will be caring for the patient to contact the physician or return to the ED if the patient

a. Vomits i. Vomiting is a sign of increasing intracranial pressure and should be reported immediately. In general, the finding of headache in the patient with a concussion is an expected abnormal observation. However, a severe headache should be reported or treated immediately. Weakness of one side of the body should be reported or treated immediately. Difficulty in waking the patient should be reported or treated immediately.

A gerontologic nurse is advocating for diagnostic testing of an 81-year-old patient who is experiencing personality changes. The nurse is aware of what factor that is known to affect the diagnosis and treatment of brain tumors in older adults? A) The effects of brain tumors are often attributed to the cognitive effects of aging. B) Brain tumors in older adults do not normally produce focal effects. C) Older adults typically have numerous benign brain tumors by the eighth decade of life. D) Brain tumors cannot normally be treated in patient over age 75.

A) The effects of brain tumors are often attributed to the cognitive effects of aging.

A patient with Huntington disease has just been admitted to a long-term care facility. The charge nurse is creating a care plan for this patient. Nutritional management for a patient with Huntington disease should be informed by what principle? A) The patient is likely to have an increased appetite. B) The patient is likely to required enzyme supplements. C) The patient will likely require a clear liquid diet. D) The patient will benefit from a low-protein diet.

A) The patient is likely to have an increased appetite.

What term is used to describe the fibrous connective tissue that hugs the brain closely and extends into every fold of the brain's surface? A) Dura mater B) Arachnoid C) Fascia D) Pia mater

Ans: D The term "meninges" describes the fibrous connective tissue that covers the brain and spinal cord. The meninges have three layers, the dura mater, arachnoid, and pia mater. The pia mater is the innermost membrane that hugs the brain closely and extends into every fold of the brain's surface. The dura mater, the outermost layer, covers the brain and spinal cord. The arachnoid, the middle membrane, is responsible for the production of cerebrospinal fluid.

The nurse is caring for a patient in the ICU who has a brain stem herniation and who is exhibiting an altered level of consciousness. Monitoring reveals that the patient's mean arterial pressure (MAP) is 60 mm Hg with an intracranial pressure (ICP) reading of 5 mm Hg. What is the nurse's most appropriate action? A) Position the patient in the high Fowler's position as tolerated. B) Administer osmotic diuretics as ordered. C) Participate in interventions to increase cerebral perfusion pressure. D) Prepare the patient for craniotomy.

C) Participate in interventions to increase cerebral perfusion pressure.

A patient with suspected Creutzfeldt-Jakob disease (CJD) is being admitted to the unit. The nurse would expect what diagnostic test to be ordered for this patient? A) Cerebral angiography B) ABG analysis C) CT D) EEG

D) EEG

A nurse in the ICU is providing care for a patient who has been admitted with a hemorrhagic stroke. The nurse is performing frequent neurologic assessments and observes that the patient is becoming progressively more drowsy over the course of the day. What is the nurse's best response to this assessment finding? A) Report this finding to the physician as an indication of decreased metabolism. B) Provide more stimulation to the patient and monitor the patient closely. C) Recognize this as the expected clinical course of a hemorrhagic stroke. D) Report this to the physician as a possible sign of clinical deterioration.

D) Report this to the physician as a possible sign of clinical deterioration.

A patient recovering from a stroke has severe shoulder pain from subluxation of the shoulder and is being cared for on the unit. To prevent further injury and pain, the nurse caring for this patient is aware of what principle of care? A) The patient should be fitted with a cast because use of a sling should be avoided due to adduction of the affected shoulder. B) Elevation of the arm and hand can lead to further complications associated with edema. C) Passively exercising the affected extremity is avoided in order to minimize pain. D) The patient should be taught to interlace fingers, place palms together, and slowly bring scapulae forward to avoid excessive force to shoulder.

D) The patient should be taught to interlace fingers, place palms together, and slowly bring scapulae forward to avoid excessive force to shoulder.

A male patient with a metastatic brain tumor is having a generalized seizure and begins vomiting. What should the nurse do first? A) Perform oral suctioning. B) Page the physician. C) Insert a tongue depressor into the patient's mouth. D) Turn the patient on his side.

D) Turn the patient on his side.

2.Which of the following conditions occurs when there is bleeding between the dura mater and arachnoid membrane?

a. Subdural hematoma

25. A client in the surgical intensive care unit has skeletal tongs in place to stabilize a cervical fracture. Protocol dictates that pin care should be performed each shift. When providing pin care for the client, which finding should the nurse report to the physician?

small amount of yellow drainage at the left pin insertion site

A client with a spinal cord injury is to receive methylprednisolone sodium succinate 100 mg intravenously twice a day. The medication is supplied in vials containing 125 mg per 2 mL. How many mL will constitute the correct dose? Enter the correct number ONLY.

1.6 (125/2=62.5 per mL) (100/62.5=1.6)

Which Glasgow Coma Scale score is indicative of a severe head injury?

7

An adult patient has sought care for the treatment of headaches that have become increasingly severe and frequent over the past several months. Which of the following questions addresses potential etiological factors? Select all that apply? A) "Are you exposed to any toxins or chemicals at work?" B) "How would you describe your ability to cope with stress?" C) "What medications are you currently taking?" D) "When was the last time you were hospitalized?" E) "Does anyone else in your family struggle with headaches?"

A) "Are you exposed to any toxins or chemicals at work?" B) "How would you describe your ability to cope with stress?" C) "What medications are you currently taking?" E) "Does anyone else in your family struggle with headaches?"

The nurse is teaching a patient with Guillain-Barré syndrome about the disease. The patient asks how he can ever recover if demyelination of his nerves is occurring. What would be the nurse's best response? A) "Guillain-Barré spares the Schwann cell, which allows for remyelination in the recovery phase of the disease." B) "In Guillain-Barré, Schwann cells replicate themselves before the disease destroys them, so remyelination is possible." C) "I know you understand that nerve cells do not remyelinate, so the physician is the best one to answer your question." D) "For some reason, in Guillain-Barré, Schwann cells become activated and take over the remyelination process."

A) "Guillain-Barré spares the Schwann cell, which allows for remyelination in the recovery phase of the disease."

An older adult has encouraged her husband to visit their primary care provider, stating that she is concerned that he may have Parkinson's disease. Which of the wife's descriptions of her husband's health and function is most suggestive of Parkinson's disease? A) "Lately he seems to move far more slowly than he ever has in the past." B) "He often complains that his joints are terribly stiff when he wakes up in the morning." C) "He's forgotten the names of some people that we've known for years." D) "He's losing weight even though he has a ravenous appetite."

A) "Lately he seems to move far more slowly than he ever has in the past."

The nurse is caring for a patient with permanent neurologic impairments resulting from a traumatic head injury. When working with this patient and family, what mutual goal should be prioritized? A) Achieve as high a level of function as possible. B) Enhance the quantity of the patient's life. C) Teach the family proper care of the patient. D) Provide community assistance.

A) Achieve as high a level of function as possible.

A patient is recovering from intracranial surgery performed approximately 24 hours ago and is complaining of a headache that the patient rates at 8 on a 10-point pain scale. What nursing action is most appropriate? A) Administer morphine sulfate as ordered. B) Reposition the patient in a prone position. C) Apply a hot pack to the patient's scalp. D) Implement distraction techniques.

A) Administer morphine sulfate as ordered.

A patient diagnosed with Bell's palsy is being cared for on an outpatient basis. During health education, the nurse should promote which of the following actions? A) Applying a protective eye shield at night B) Chewing on the affected side to prevent unilateral neglect C) Avoiding the use of analgesics whenever possible D) Avoiding brushing the teeth

A) Applying a protective eye shield at night

The nurse is caring for a patient whose recent health history includes an altered LOC. What should be the nurse's first action when assessing this patient? A) Assessing the patient's verbal response B) Assessing the patient's ability to follow complex commands C) Assessing the patient's judgment D) Assessing the patient's response to pain

A) Assessing the patient's verbal response

A patient who just suffered a suspected ischemic stroke is brought to the ED by ambulance. On what should the nurse's primary assessment focus? A) Cardiac and respiratory status B) Seizure activity C) Pain D) Fluid and electrolyte balance

A) Cardiac and respiratory status

A nurse is planning discharge education for a patient who underwent a cervical diskectomy. What strategies would the nurse assess that would aid in planning discharge teaching? A) Care of the cervical collar B) Technique for performing neck ROM exercises C) Home assessment of ABGs D) Techniques for restoring nerve function

A) Care of the cervical collar

The nurse caring for a patient in a persistent vegetative state is regularly assessing for potential complications. Complications of neurologic dysfunction for which the nurse should assess include which of the following? Select all that apply. A) Contractures B) Hemorrhage C) Pressure ulcers D) Venous thromboembolism E) Pneumonia

A) Contractures C) Pressure ulcers D) Venous thromboembolism E) Pneumonia

The nurse has created a plan of care for a patient who is at risk for increased ICP. The patient's care plan should specify monitoring for what early sign of increased ICP? A) Disorientation and restlessness B) Decreased pulse and respirations C) Projectile vomiting D) Loss of corneal reflex

A) Disorientation and restlessness

The nurse is caring for a patient who is hospitalized with an exacerbation of MS. To ensure the patient's safety, what nursing action should be performed? A) Ensure that suction apparatus is set up at the bedside. B) Pad the patient's bed rails. C) Maintain bed rest whenever possible. D) Provide several small meals each day.

A) Ensure that suction apparatus is set up at the bedside.

A patient with MS has been admitted to the hospital following an acute exacerbation. When planning the patient's care, the nurse addresses the need to enhance the patient's bladder control. What aspect of nursing care is most likely to meet this goal? A) Establish a timed voiding schedule. B) Avoid foods that change the pH of urine. C) Perform intermittent catheterization q6h. D) Administer anticholinergic drugs as ordered.

A) Establish a timed voiding schedule.

A patient is brought by ambulance to the ED after suffering what the family thinks is a stroke. The nurse caring for this patient is aware that an absolute contraindication for thrombolytic therapy is what? A) Evidence of hemorrhagic stroke B) Blood pressure of 180/110 mm Hg C) Evidence of stroke evolution D) Previous thrombolytic therapy within the past 12 months

A) Evidence of hemorrhagic stroke

The nurse is assessing a patient with a suspected stroke. What assessment finding is most suggestive of a stroke? A) Facial droop B) Dysrhythmias C) Periorbital edema D) Projectile vomiting

A) Facial droop

When caring for a patient with increased ICP the nurse knows the importance of monitoring for possible secondary complications, including syndrome of inappropriate antidiuretic hormone (SIADH). What nursing interventions would the nurse most likely initiate if the patient developed SIADH? A) Fluid restriction B) Transfusion of platelets C) Transfusion of fresh frozen plasma (FFP) D) Electrolyte restriction

A) Fluid restriction

When preparing to discharge a patient home, the nurse has met with the family and warned them that the patient may exhibit unexpected emotional responses. The nurse should teach the family that these responses are typically a result of what cause? A) Frustration around changes in function and communication B) Unmet physiologic needs C) Changes in brain activity during sleep and wakefulness D) Temporary changes in metabolism

A) Frustration around changes in function and communication

A patient has been admitted to the neurologic ICU with a diagnosis of a brain tumor. The patient is scheduled to have a tumor resection/removal in the morning. Which of the following assessment parameters should the nurse include in the initial assessment? A) Gag reflex B) Deep tendon reflexes C) Abdominal girth D) Hearing acuity

A) Gag reflex

The nurse is caring for a patient who sustained a moderate head injury following a bicycle accident. The nurse's most recent assessment reveals that the patient's respiratory effort has increased. What is the nurse's most appropriate response? A) Inform the care team and assess for further signs of possible increased ICP. B) Administer bronchodilators as ordered and monitor the patient's LOC. C) Increase the patient's bed height and reassess in 30 minutes. D) Administer a bolus of normal saline as ordered.

A) Inform the care team and assess for further signs of possible increased ICP.

A family member of a patient diagnosed with Huntington disease calls you at the clinic. She is requesting help from the Huntington's Disease Society of America. What kind of help can this patient and family receive from this organization? Select all that apply. A) Information about this disease B) Referrals C) Public education D) Individual assessments E) Appraisals of research studies

A) Information about this disease B) Referrals C) Public education

A 35-year-old woman is diagnosed with a peripheral neuropathy. When making her plan of care, the nurse knows to include what in patient teaching? Select all that apply. A) Inspect the lower extremities for skin breakdown. B) Footwear needs to be accurately sized. C) Immediate family members should be screened for the disease. D) Assistive devices may be needed to reduce the risk of falls. E) Dietary modifications are likely necessary.

A) Inspect the lower extremities for skin breakdown. B) Footwear needs to be accurately sized. D) Assistive devices may be needed to reduce the risk of falls.

A nurse is assessing a patient with an acoustic neuroma who has been recently admitted to an oncology unit. What symptoms is the nurse likely to find during the initial assessment? A) Loss of hearing, tinnitus, and vertigo B) Loss of vision, change in mental status, and hyperthermia C) Loss of hearing, increased sodium retention, and hypertension D) Loss of vision, headache, and tachycardia

A) Loss of hearing, tinnitus, and vertigo

The nurse is working with a patient who is newly diagnosed with MS. What basic information should the nurse provide to the patient? A) MS is a progressive demyelinating disease of the nervous system. B) MS usually occurs more frequently in men. C) MS typically has an acute onset. D) MS is sometimes caused by a bacterial infection.

A) MS is a progressive demyelinating disease of the nervous system.

During a patient's recovery from stroke, the nurse should be aware of predictors of stroke outcome in order to help patients and families set realistic goals. What are the predictors of stroke outcome? Select all that apply. A) National Institutes of Health Stroke Scale (NIHSS) score B) Race C) LOC at time of admission D) Gender E) Age

A) National Institutes of Health Stroke Scale (NIHSS) score C) LOC at time of admission E) Age

A patient with an inoperable brain tumor has been told that he has a short life expectancy. On what aspects of assessment and care should the home health nurse focus? Select all that apply. A) Pain control B) Management of treatment complications C) Interpretation of diagnostic tests D) Assistance with self-care E) Administration of treatments

A) Pain control B) Management of treatment complications D) Assistance with self-care E) Administration of treatments

A hospital patient has experienced a seizure. In the immediate recovery period, what action best protects the patient's safety? A) Place the patient in a side-lying position. B) Pad the patient's bed rails. C) Administer antianxiety medications as ordered. D) Reassure the patient and family members.

A) Place the patient in a side-lying position.

After a major ischemic stroke, a possible complication is cerebral edema. Nursing care during the immediate recovery period from an ischemic stroke should include which of the following? A) Positioning to avoid hypoxia B) Maximizing PaCO2 C) Administering hypertonic IV solution D) Initiating early mobilization

A) Positioning to avoid hypoxia

The nurse is caring for a 77-year-old woman with MS. She states that she is very concerned about the progress of her disease and what the future holds. The nurse should know that elderly patients with MS are known to be particularly concerned about what variables? Select all that apply. A) Possible nursing home placement B) Pain associated with physical therapy C) Increasing disability D) Becoming a burden on the family E) Loss of appetite

A) Possible nursing home placement C) Increasing disability D) Becoming a burden on the family

A male patient presents at the free clinic with complaints of impotency. Upon physical examination, the nurse practitioner notes the presence of hypogonadism. What diagnosis should the nurse suspect? A) Prolactinoma B) Angioma C) Glioma D) Adrenocorticotropic hormone (ACTH)-producing adenoma

A) Prolactinoma

The nurse caring for a patient diagnosed with Parkinson's disease has prepared a plan of care that would include what goal? A) Promoting effective communication B) Controlling diarrhea C) Preventing cognitive decline D) Managing choreiform movements

A) Promoting effective communication

The patient has been diagnosed with aphasia after suffering a stroke. What can the nurse do to best make the patient's atmosphere more conducive to communication? A) Provide a board of commonly used needs and phrases. B) Have the patient speak to loved ones on the phone daily. C) Help the patient complete his or her sentences. D) Speak in a loud and deliberate voice to the patient.

A) Provide a board of commonly used needs and phrases.

A patient is postoperative day 1 following intracranial surgery. The nurse's assessment reveals that the patient's LOC is slightly decreased compared with the day of surgery. What is the nurse's best response to this assessment finding? A) Recognize that this may represent the peak of post-surgical cerebral edema. B) Alert the surgeon to the possibility of an intracranial hemorrhage. C) Understand that the surgery may have been unsuccessful. D) Recognize the need to refer the patient to the palliative care team.

A) Recognize that this may represent the peak of post-surgical cerebral edema.

Following diagnostic testing, a patient has been admitted to the ICU and placed on cerebral aneurysm precautions. What nursing action should be included in patient's plan of care? A) Supervise the patient's activities of daily living closely. B) Initiate early ambulation to prevent complications of immobility. C) Provide a high-calorie, low-protein diet. D) Perform all of the patient's hygiene and feeding.

A) Supervise the patient's activities of daily living closely.

A patient who has experienced an ischemic stroke has been admitted to the medical unit. The patient's family in adamant that she remain on bed rest to hasten her recovery and to conserve energy. What principle of care should inform the nurse's response to the family? A) The patient should mobilize as soon as she is physically able. B) To prevent contractures and muscle atrophy, bed rest should not exceed 4 weeks. C) The patient should remain on bed rest until she expresses a desire to mobilize. D) Lack of mobility will greatly increase the patient's risk of stroke recurrence.

A) The patient should mobilize as soon as she is physically able.

The nurse is admitting a patient to the unit who is scheduled for removal of an intracranial mass. What diagnostic procedures might be included in this patient's admission orders? Select all that apply. A) Transcranial Doppler flow study B) Cerebral angiography C) MRI D) Cranial radiography E) Electromyelography (EMG)

A) Transcranial Doppler flow study B) Cerebral angiography C) MRI

A patient is scheduled for a myelogram and the nurse explains to the patient that this is an invasive procedure, which assesses for any lesions in the spinal cord. The nurse should explain that the preparation is similar to which of the following neurologic tests? A) Lumbar puncture B) MRI C) Cerebral angiography D) EEG

Ans: A Feedback: A myelogram is an x-ray of the spinal subarachnoid space taken after the injection of a contrast agent into the spinal subarachnoid space through a lumbar puncture. Patient preparation for a myelogram would be similar to that for lumbar puncture. The other listed diagnostic tests do not involve lumbar puncture.

A patient with lower back pain is scheduled for myelography using metrizamide (a water-soluble contrast dye). After the test, the nurse should prioritize what action? A) Positioning the patient with the head of the bed elevated 45 degrees B) Administering IV morphine sulfate to prevent headache C) Limiting fluids for the next 12 hours D) Helping the patient perform deep breathing and coughing exercises

Ans: A Feedback: After myelography, the patient lies in bed with the head of the bed elevated 30 to 45 degrees. The patient is advised to remain in bed in the recommended position for 3 hours or as prescribed. Drinking liberal amounts of fluid for rehydration and replacement of CSF may decrease the incidence of post-lumbar puncture headache. Deep breathing and coughing exercises are not normally necessary since there is no consequent risk of atelectasis.

A patient is being given a medication that stimulates her parasympathetic system. Following administration of this medication, the nurse should anticipate what effect? A) Constricted pupils B) Dilated bronchioles C) Decreased peristaltic movement D) Relaxed muscular walls of the urinary bladder

Ans: A Feedback: Parasympathetic stimulation results in constricted pupils, constricted bronchioles, increased peristaltic movement, and contracted muscular walls of the urinary bladder.

A gerontologic nurse educator is providing practice guidelines to unlicensed care providers. Because reaction to painful stimuli is sometimes blunted in older adults, what must be used with caution? A) Hot or cold packs B) Analgesics C) Anti-inflammatory medications D) Whirlpool baths

Ans: A Feedback: Reaction to painful stimuli may be decreased with age. Because pain is an important warning signal, caution must be used when hot or cold packs are used. The older patient may be burned or suffer frostbite before being aware of any discomfort. Any medication is used with caution in the elderly, but not because of the decreased sense of heat or cold. Whirlpool baths are generally not a routine treatment ordered for the elderly.

A patient exhibiting an uncoordinated gait has presented at the clinic. Which of the following is the most plausible cause of this patient's health problem? A) Cerebellar dysfunction B) A lesion in the pons C) Dysfunction of the medulla D) A hemorrhage in the midbrain

Ans: A Feedback: The cerebellum controls fine movement, balance, position sense, and integration of sensory input. Portions of the pons control the heart, respiration, and blood pressure. Cranial nerves IX through XII connect to the brain in the medulla. Cranial nerves III and IV originate in the midbrain.

A patient is admitted to the medical unit with an exacerbation of multiple sclerosis. When assessing this patient, the nurse has the patient stick out her tongue and move it back and forth. What is the nurse assessing? A) Function of the hypoglossal nerve B) Function of the vagus nerve C) Function of the spinal nerve D) Function of the trochlear nerve

Ans: A Feedback: The hypoglossal nerve is the 12th cranial nerve. It is responsible for movement of the tongue. None of the other listed nerves affects motor function in the tongue.

The patient in the ED has just had a diagnostic lumbar puncture. To reduce the incidence of a post-lumbar puncture headache, what is the nurse's most appropriate action? A) Position the patient prone. B) Position the patient supine with the head of bed flat. C) Position the patient left side-lying. D) Administer acetaminophen as ordered.

Ans: A The lumbar puncture headache may be avoided if a small-gauge needle is used and if the patient remains prone after the procedure. Acetaminophen is not administered as a preventative measure for post-lumbar puncture headaches.

The nurse is doing an initial assessment on a patient newly admitted to the unit with a diagnosis of cerebrovascular accident (CVA). The patient has difficulty copying a figure that the nurse has drawn and is diagnosed with visual-receptive aphasia. What brain region is primarily involved in this deficit? A) Temporal lobe B) Parietal-occipital area C) Inferior posterior frontal areas D) Posterior frontal area

Ans: B Difficulty copying a figure that the nurse has drawn would be considered visual-receptive aphasia, which involves the parietal-occipital area. Expressive aphasia, the inability to express oneself, is often associated with damage to the frontal area. Receptive aphasia, the inability to understand what someone else is saying, is often associated with damage to the temporal lobe area.

The nurse is caring for a patient who exhibits abnormal results of the Weber test and Rinne test. The nurse should suspect dysfunction involving what cranial nerve? A) Trigeminal B) Acoustic C) Hypoglossal D) Trochlear

Ans: B Feedback: Abnormal hearing can correlate with damage to cranial nerve VIII (acoustic). The acoustic nerve functions in hearing and equilibrium. The trigeminal nerve functions in facial sensation, corneal reflex, and chewing. The hypoglossal nerve moves the tongue. The trochlear nerve controls muscles that move the eye.

The nurse is performing a neurologic assessment of a patient whose injuries have rendered her unable to follow verbal commands. How should the nurse proceed with assessing the patient's level of consciousness (LOC)? A) Assess the patient's vital signs and correlate these with the patient's baselines. B) Assess the patient's eye opening and response to stimuli. C) Document that the patient currently lacks a level of consciousness. D) Facilitate diagnostic testing in an effort to obtain objective data.

Ans: B Feedback: If the patient is not alert or able to follow commands, the examiner observes for eye opening; verbal response and motor response to stimuli, if any; and the type of stimuli needed to obtain a response. Vital signs and diagnostic testing are appropriate, but neither will allow the nurse to gauge the patient's LOC. Inability to follow commands does not necessarily denote an absolute lack of consciousness.

In the course of a focused neurologic assessment, the nurse is palpating the patient's major muscle groups at rest and during passive movement. Data gleaned from this assessment will allow the nurse to describe which of the following aspects of neurologic function? A) Muscle dexterity B) Muscle tone C) Motor symmetry D) Deep tendon reflexes

Ans: B Feedback: Muscle tone (the tension present in a muscle at rest) is evaluated by palpating various muscle groups at rest and during passive movement. Data from this assessment do not allow the nurse to ascertain the patient's dexterity, reflexes, or motor symmetry

The nurse is planning the care of a patient with Parkinson's disease. The nurse should be aware that treatment will focus on what pathophysiological phenomenon? A) Premature degradation of acetylcholine B) Decreased availability of dopamine C) Insufficient synthesis of epinephrine D) Delayed reuptake of serotonin

Ans: B Feedback: Parkinson's disease develops from decreased availability of dopamine, not acetylcholine, epinephrine, or serotonin.

The physician has ordered a somatosensory evoked responses (SERs) test for a patient for whom the nurse is caring. The nurse is justified in suspecting that this patient may have a history of what type of neurologic disorder? A) Hypothalamic disorder B) Demyelinating disease C) Brainstem deficit D) Diabetic neuropathy

Ans: B Feedback: SERs are used to detect deficits in the spinal cord or peripheral nerve conduction and to monitor spinal cord function during surgical procedures. The test is also useful in the diagnosis of demyelinating diseases, such as multiple sclerosis and polyneuropathies, where nerve conduction is slowed. The test is not done to diagnose hypothalamic disorders, brainstem deficits, or diabetic neuropathies.

A trauma patient in the ICU has been declared brain dead. What diagnostic test is used in making the determination of brain death? A) Magnetic resonance imaging (MRI) B) Electroencephalography (EEG) C) Electromyelography (EMG) D) Computed tomography (CT

Ans: B Feedback: The EEG can be used in determining brain death. MRI, CT, and EMG are not normally used in determining brain death.

Assessment is crucial to the care of patients with neurologic dysfunction. What does accurate and appropriate assessment require? Select all that apply. A) The ability to select mediations for the neurologic dysfunction B) Understanding of the tests used to diagnose neurologic disorders C) Knowledge of nursing interventions related to assessment and diagnostic testing D) Knowledge of the anatomy of the nervous system E) The ability to interpret the results of diagnostic tests

Ans: B, C, D Feedback: Assessment requires knowledge of the anatomy and physiology of the nervous system and an understanding of the array of tests and procedures used to diagnose neurologic disorders. Knowledge about the nursing implications and interventions related to assessment and diagnostic testing is also essential. Selecting medications and interpreting diagnostic tests are beyond the normal scope of the nurse.

The nurse educator is reviewing the assessment of cranial nerves. What should the educator identify as the specific instances when cranial nerves should be assessed? Select all that apply. A) When a neurogenic bladder develops B) When level of consciousness is decreased C) With brain stem pathology D) In the presence of peripheral nervous system disease E) When a spinal reflex is interrupted

Ans: B, C, D Feedback: Cranial nerves are assessed when level of consciousness is decreased, with brain stem pathology, or in the presence of peripheral nervous system disease. Abnormalities in muscle tone and involuntary movements are less likely to prompt the assessment of cranial nerves, since these nerves do not directly mediate most aspects of muscle tone and movement.

A patient had a lumbar puncture performed at the outpatient clinic and the nurse has phoned the patient and family that evening. What does this phone call enable the nurse to determine? A) What are the patient's and family's expectations of the test B) Whether the patient's family had any questions about why the test was necessary C) Whether the patient has had any complications of the test D) Whether the patient understood accurately why the test was done

Ans: C Feedback: Contacting the patient and family after diagnostic testing enables the nurse to determine whether they have any questions about the procedure or whether the patient had any untoward results. The other listed information should have been elicited from the patient and family prior to the test.

When caring for a patient with an altered level of consciousness, the nurse is preparing to test cranial nerve VII. What assessment technique would the nurse use to elicit a response from cranial nerve VII? A) Palpate trapezius muscle while patient shrugs should against resistance. B) Administer the whisper or watch-tick test. C) Observe for facial movement symmetry, such as a smile. D) Note any hoarseness in the patient's voice.

Ans: C Feedback: Cranial nerve VII is the facial nerve. An appropriate assessment technique for this cranial nerve would include observing for symmetry while the patient performs facial movements: smiles, whistles, elevates eyebrows, and frowns. Palpating and noting strength of the trapezius muscle while the patient shrugs shoulders against resistance would be completed to assess cranial nerve XI (spinal accessory). Assessing cranial nerve VIII (acoustic) would involve using the whisper or watch-tick test to evaluate hearing. Noting any hoarseness in the patient's voice would involve assessment of cranial nerve X (vagus)

A patient is scheduled for CT scanning of the head because of a recent onset of neurologic deficits. What should the nurse tell the patient in preparation for this test? A) "No metal objects can enter the procedure room." B) "You need to fast for 8 hours prior to the test." C) "You will need to lie still throughout the procedure." D) "There will be a lot of noise during the test."

Ans: C Feedback: Preparation for CT scanning includes teaching the patient about the need to lie quietly throughout the procedure. If the patient were having an MRI, metal and noise would be appropriate teaching topics. There is no need to fast prior to a CT scan of the brain.

During the performance of the Romberg test, the nurse observes that the patient sways slightly. What is the nurse's most appropriate action? A) Facilitate a referral to a neurologist. B) Reposition the patient supine to ensure safety. C) Document successful completion of the assessment. D) Follow up by having the patient perform the Rinne test.

Ans: C Feedback: Slight swaying during the Romberg test is normal, but a loss of balance is abnormal and is considered a positive Romberg test. Slight swaying is not a significant threat to the patient's safety. The Rinne test assesses hearing, not balance.

A patient is having a "fight or flight response" after receiving bad news about his prognosis. What affect will this have on the patient's sympathetic nervous system? A) Constriction of blood vessels in the heart muscle B) Constriction of bronchioles C) Increase in the secretion of sweat D) Constriction of pupils

Ans: C Feedback: Sympathetic nervous system stimulation results in dilated blood vessels in the heart and skeletal muscle, dilated bronchioles, increased secretion of sweat, and dilated pupils.

A trauma patient was admitted to the ICU with a brain injury. The patient had a change in level of consciousness, increased vital signs, and became diaphoretic and agitated. The nurse should recognize which of the following syndromes as the most plausible cause of these symptoms? A) Adrenal crisis B) Hypothalamic collapse C) Sympathetic storm D) Cranial nerve deficit

Ans: C Feedback: Sympathetic storm is a syndrome associated with changes in level of consciousness, altered vital signs, diaphoresis, and agitation that may result from hypothalamic stimulation of the sympathetic nervous system following traumatic brain injury. Alterations in cranial nerve or adrenal function would not have this result.

The nurse caring for an 80 year-old patient knows that she has a pre-existing history of dulled tactile sensation. The nurse should first consider what possible cause for this patient's diminished tactile sensation? A) Damage to cranial nerve VIII B) Adverse medication effects C) Age-related neurologic changes D) An undiagnosed cerebrovascular accident in early adulthood

Ans: C Feedback: Tactile sensation is dulled in the elderly person due to a decrease in the number of sensory receptors. While thorough assessment is necessary, it is possible that this change is unrelated to pathophysiological processes.

A patient in the OR goes into malignant hyperthermia due to an abnormal reaction to the anesthetic. The nurse knows that the area of the brain that regulates body temperature is which of the following? A) Cerebellum B) Thalamus C) Hypothalamus D) Midbrain

Ans: C Feedback: The hypothalamus plays an important role in the endocrine system because it regulates the pituitary secretion of hormones that influence metabolism, reproduction, stress response, and urine production. It works with the pituitary to maintain fluid balance through hormonal release and maintains temperature regulation by promoting vasoconstriction or vasodilatation. The cerebellum, thalamus, and midbrain and not directly involved in temperature regulation.

The nurse is caring for a patient with an upper motor neuron lesion. What clinical manifestations should the nurse anticipate when planning the patient's neurologic assessment? A) Decreased muscle tone B) Flaccid paralysis C) Loss of voluntary control of movement D) Slow reflexes

Ans: C Upper motor neuron lesions do not cause muscle atrophy, flaccid paralysis, or slow reflexes. However, upper motor neuron lesions normally cause loss of voluntary control.

A 72-year-old man has been brought to his primary care provider by his daughter, who claims that he has been experiencing uncharacteristic lapses in memory. What principle should underlie the nurse's assessment and management of this patient? A) Loss of short-term memory is normal in older adults, but loss of long-term memory is pathologic. B) Lapses in memory in older adults are considered benign unless they have negative consequences. C) Gradual increases in confusion accompany the aging process. D) Thorough assessment is necessary because changes in cognition are always considered to be pathologic

Ans: D Feedback: Although mental processing time decreases with age, memory, language, and judgment capacities remain intact. Change in mental status should never be assumed to be a normal part of aging.

The neurologic nurse is testing the function of a patient's cerebellum and basal ganglia. What action will most accurately test these structures? A) Have the patient identify the location of a cotton swab on his or her skin with the eyes closed. B) Elicit the patient's response to a hypothetical problem. C) Ask the patient to close his or her eyes and discern between hot and cold stimuli. D) Guide the patient through the performance of rapid, alternating movements.

Ans: D Feedback: Cerebellar and basal ganglia influence on the motor system is reflected in balance control and coordination. Coordination in the hands and upper extremities is tested by having the patient perform rapid, alternating movements and point-to-point testing. The cerebellum and basal ganglia do not mediate cutaneous sensation or judgment.

A nurse is caring for a patient diagnosed with Ménière's disease. While completing a neurologic examination on the patient, the nurse assesses cranial nerve VIII. The nurse would be correct in identifying the function of this nerve as what? A) Movement of the tongue B) Visual acuity C) Sense of smell D) Hearing and equilibrium

Ans: D Feedback: Cranial nerve VIII (acoustic) is responsible for hearing and equilibrium. Cranial nerve XII (hypoglossal) is responsible for movement of the tongue. Cranial nerve II (optic) is responsible for visual acuity and visual fields. Cranial nerve I (olfactory) functions in sense of smell.

The nursing students are learning how to assess function of cranial nerve VIII. To assess the function of cranial nerve VIII the students would be correct in completing which of the following assessment techniques? A) Have the patient identify familiar odors with the eyes closed. B) Assess papillary reflex. C) Utilize the Snellen chart. D) Test for air and bone conduction (Rinne test).

Ans: D Feedback: Cranial nerve VIII is the acoustic nerve. It functions in hearing and equilibrium. When assessing this nerve, the nurse would test for air and bone conduction (Rinne) with a tuning fork. Assessment of papillary reflex would be completed for cranial nerves III (oculomotor), IV (trochlear), and VI (abducens). The Snellen chart would be used to assess cranial nerve II (optic).

A patient for whom the nurse is caring has positron emission tomography (PET) scheduled. In preparation, what should the nurse explain to the patient? A) The test will temporarily limit blood flow through the brain. B) An allergy to iodine precludes getting the radio-opaque dye. C) The patient will need to endure loud noises during the test. D) The test may result in dizziness or lightheadedness.

Ans: D Feedback: Key nursing interventions for PET scan include explaining the test and teaching the patient about inhalation techniques and the sensations (e.g., dizziness, light-headedness, and headache) that may occur. A PET scan does not impede blood flow through the brain. An allergy to iodine precludes the dye for an MRI, and loud noise is heard in an MRI.

The nurse is conducting a focused neurologic assessment. When assessing the patient's cranial nerve function, the nurse would include which of the following assessments? A) Assessment of hand grip B) Assessment of orientation to person, time, and place C) Assessment of arm drift D) Assessment of gag reflex

Ans: D Feedback: The gag reflex is governed by the glossopharyngeal nerve, one of the cranial nerves. Hand grip and arm drifting are part of motor function assessment. Orientation is an assessment parameter related to a mental status examination.

A patient is currently being stimulated by the parasympathetic nervous system. What effect will this nervous stimulation have on the patient's bladder? A) The parasympathetic nervous system causes urinary retention. B) The parasympathetic nervous system causes bladder spasms. C) The parasympathetic nervous system causes urge incontinence. D) The parasympathetic nervous system makes the bladder contract.

Ans: D Feedback: The parasympathetic division of the nervous system causes contraction (stimulation) of the urinary bladder muscles and a decrease (inhibition) in heart rate, whereas the sympathetic division produces relaxation (inhibition) of the urinary bladder and an increase (stimulation) in the rate and force of the heartbeat.

The nurse is admitting a patient to the unit who is diagnosed with a lower motor neuron lesion. What entry in the patient's electronic record is most consistent with this diagnosis? A) "Patient exhibits increased muscle tone." B) "Patient demonstrates normal muscle structure with no evidence of atrophy." C) "Patient demonstrates hyperactive deep tendon reflexes." D) "Patient demonstrates an absence of deep tendon reflexes."

Ans: D Lower motor neuron lesions cause flaccid muscle paralysis, muscle atrophy, decreased muscle tone, and loss of voluntary control.

An elderly patient is being discharged home. The patient lives alone and has atrophy of his olfactory organs. The nurse tells the patient's family that it is essential that the patient have what installed in the home? A) Grab bars B) Nonslip mats C) Baseboard heaters D) A smoke detector

Ans: D The sense of smell deteriorates with age. The olfactory organs are responsible for smell. This may present a safety hazard for the patient because he or she may not smell smoke or gas leaks. Smoke detectors are universally necessary, but especially for this patient.

A nurse is caring for a patient diagnosed with a hemorrhagic stroke. When creating this patient's plan of care, what goal should be prioritized? A) Prevent complications of immobility. B) Maintain and improve cerebral tissue perfusion. C) Relieve anxiety and pain. D) Relieve sensory deprivation.

B) Maintain and improve cerebral tissue perfusion.

A patient scheduled for magnetic resonance imaging (MRI) has arrived at the radiology department. The nurse who prepares the patient for the MRI should prioritize which of the following actions? A) Withholding stimulants 24 to 48 hours prior to exam B) Removing all metal-containing objects C) Instructing the patient to void prior to the MRI D) Initiating an IV line for administration of contrast

B Patient preparation for an MRI consists of removing all metal-containing objects prior to the examination. Withholding stimulants would not affect an MRI; this relates to an electroencephalography (EEG). Instructing the patient to void is patient preparation for a lumbar puncture. Initiating an IV line for administration of contrast would be done if the patient was having a CT scan with contrast.

A gerontologic nurse planning the neurologic assessment of an older adult is considering normal, age-related changes. Of what phenomenon should the nurse be aware? A) Hyperactive deep tendon reflexes B) Reduction in cerebral blood flow C) Increased cerebral metabolism D) Hypersensitivity to painful stimuli

B Reduction in cerebral blood flow (CBF) is a change that occurs in the normal aging process. Deep tendon reflexes can be decreased or, in some cases, absent. Cerebral metabolism decreases as the patient advances in age. Reaction to painful stimuli may be decreased with age. Because pain is an important warning signal, caution must be used when hot or cold packs are used.

The nurse has admitted a new patient to the unit. One of the patient's admitting orders is for an adrenergic medication. The nurse knows that this medication will have what effect on the circulatory system? A) Thin, watery saliva B) Increased heart rate C) Decreased BP D) Constricted bronchioles

B The term "adrenergic" refers to the sympathetic nervous system. Sympathetic effects include an increased rate and force of the heartbeat. Cholinergic effects, which correspond to the parasympathetic division of the autonomic nervous system, include thin, watery saliva, decreased rate and force of heartbeat, and decreased BP.

A family member brings the patient to the clinic for a follow-up visit after a stroke. The family member asks the nurse what he can do to decrease his chance of having another stroke. What would be the nurse's best answer? A) "Have your heart checked regularly." B) "Stop smoking as soon as possible." C) "Get medication to bring down your sodium levels." D) "Eat a nutritious diet."

B) "Stop smoking as soon as possible."

A rehabilitation nurse caring for a patient who has had a stroke is approached by the patient's family and asked why the patient has to do so much for herself when she is obviously struggling. What would be the nurse's best answer? A) "We are trying to help her be as useful as she possibly can." B) "The focus on care in a rehabilitation facility is to help the patient to resume as much self-care as possible." C) "We aren't here to care for her the way the hospital staff did; we are here to help her get better so she can go home." D) "Rehabilitation means helping patients do exactly what they did before their stroke."

B) "The focus on care in a rehabilitation facility is to help the patient to resume as much self-care as possible."

A school nurse is called to the playground where a 6-year-old girl has been found unresponsive and "staring into space," according to the playground supervisor. How would the nurse document the girl's activity in her chart at school? A) Generalized seizure B) Absence seizure C) Focal seizure D) Unclassified seizure

B) Absence seizure

A patient with herpes simplex virus encephalitis (HSV) has been admitted to the ICU. What medication would the nurse expect the physician to order for the treatment of this disease process? A) Cyclosporine (Neoral) B) Acyclovir (Zovirax) C) Cyclobenzaprine (Flexeril) D) Ampicillin (Prinicpen)

B) Acyclovir (Zovirax)

A clinic nurse is caring for a patient diagnosed with migraine headaches. During the patient teaching session, the patient questions the nurse regarding alcohol consumption. What would the nurse be correct in telling the patient about the effects of alcohol? A) Alcohol causes hormone fluctuations. B) Alcohol causes vasodilation of the blood vessels. C) Alcohol has an excitatory effect on the CNS. D) Alcohol diminishes endorphins in the brain.

B) Alcohol causes vasodilation of the blood vessels.

When caring for a patient who had a hemorrhagic stroke, close monitoring of vital signs and neurologic changes is imperative. What is the earliest sign of deterioration in a patient with a hemorrhagic stroke of which the nurse should be aware? A) Generalized pain B) Alteration in level of consciousness (LOC) C) Tonic-clonic seizures D) Shortness of breath

B) Alteration in level of consciousness (LOC)

A nurse is caring for a patient who experiences debilitating cluster headaches. The patient should be taught to take appropriate medications at what point in the course of the onset of a new headache? A) As soon as the patient's pain becomes unbearable B) As soon as the patient senses the onset of symptoms C) Twenty to 30 minutes after the onset of symptoms D) When the patient senses his or her symptoms peaking

B) As soon as the patient senses the onset of symptoms

A patient diagnosed with myasthenia gravis has been hospitalized to receive plasmapheresis for a myasthenic exacerbation. The nurse knows that the course of treatment for plasmapheresis in a patient with myasthenia gravis is what? A) Every day for 1 week B) Determined by the patient's response C) Alternate days for 10 days D) Determined by the patient's weight

B) Determined by the patient's response

The clinic nurse caring for a patient with Parkinson's disease notes that the patient has been taking levodopa and carbidopa (Sinemet) for 7 years. For what common side effect of Sinemet would the nurse assesses this patient? A) Pruritus B) Dyskinesia C) Lactose intolerance D) Diarrhea

B) Dyskinesia

When caring for a patient who has had a stroke, a priority is reduction of ICP. What patient position is most consistent with this goal? A) Head turned slightly to the right side B) Elevation of the head of the bed C) Position changes every 15 minutes while awake D) Extension of the neck

B) Elevation of the head of the bed

A 25-year-old female patient with brain metastases is considering her life expectancy after her most recent meeting with her oncologist. Based on the fact that the patient is not receiving treatment for her brain metastases, what is the nurse's most appropriate action? A) Promoting the patient's functional status and ADLs B) Ensuring that the patient receives adequate palliative care C) Ensuring that the family does not tell the patient that her condition is terminal D) Promoting adherence to the prescribed medication regimen

B) Ensuring that the patient receives adequate palliative care

The nurse is developing a plan of care for a patient newly diagnosed with Bell's palsy. The nurse's plan of care should address what characteristic manifestation of this disease? A) Tinnitus B) Facial paralysis C) Pain at the base of the tongue D) Diplopia

B) Facial paralysis

A patient exhibiting an altered level of consciousness (LOC) due to blunt-force trauma to the head is admitted to the ED. The physician determines the patient's injury is causing increased intracranial pressure (ICP). The nurse should gauge the patient's LOC on the results of what diagnostic tool? A) Monro-Kellie hypothesis B) Glasgow Coma Scale C) Cranial nerve function D) Mental status examination

B) Glasgow Coma Scale

A 69-year-old patient is brought to the ED by ambulance because a family member found him lying on the floor disoriented and lethargic. The physician suspects bacterial meningitis and admits the patient to the ICU. The nurse knows that risk factors for an unfavorable outcome include what? Select all that apply. A) Blood pressure greater than 140/90 mm Hg B) Heart rate greater than 120 bpm C) Older age D) Low Glasgow Coma Scale E) Lack of previous immunizations

B) Heart rate greater than 120 bpm C) Older age D) Low Glasgow Coma Scale

A patient has been admitted to the neurologic unit for the treatment of a newly diagnosed brain tumor. The patient has just exhibited seizure activity for the first time. What is the nurse's priority response to this event? A) Identify the triggers that precipitated the seizure. B) Implement precautions to ensure the patient's safety. C) Teach the patient's family about the relationship between brain tumors and seizure activity. D) Ensure that the patient is housed in a private room.

B) Implement precautions to ensure the patient's safety.

A patient has just returned to the unit from the PACU after surgery for a tumor within the spine. The patient complains of pain. When positioning the patient for comfort and to reduce injury to the surgical site, the nurse will position to patient in what position? A) In the high Fowler's position B) In a flat side-lying position C) In the Trendelenberg position D) In the reverse Trendelenberg position

B) In a flat side-lying position

The nurse is caring for a patient who is in status epilepticus. What medication does the nurse know may be given to halt the seizure immediately? A) Intravenous phenobarbital (Luminal) B) Intravenous diazepam (Valium) C) Oral lorazepam (Ativan) D) Oral phenytoin (Dilantin)

B) Intravenous diazepam (Valium)

The nurse is participating in the care of a patient with increased ICP. What diagnostic test is contraindicated in this patient's treatment? A) Computed tomography (CT) scan B) Lumbar puncture C) Magnetic resonance imaging (MRI) D) Venous Doppler studies

B) Lumbar puncture

The neurologic ICU nurse is admitting a patient following a craniotomy using the supratentorial approach. How should the nurse best position the patient? A) Position the patient supine. B) Maintain head of bed (HOB) elevated at 30 to 45 degrees. C) Position patient in prone position. D) Maintain bed in Trendelenberg position.

B) Maintain head of bed (HOB) elevated at 30 to 45 degrees.

A patient has been admitted to the ICU after being recently diagnosed with an aneurysm and the patient's admission orders include specific aneurysm precautions. What nursing action will the nurse incorporate into the patient's plan of care? A) Elevate the head of the bed to 45 degrees. B) Maintain the patient on complete bed rest. C) Administer enemas when the patient is constipated. D) Avoid use of thigh-high elastic compression stockings.

B) Maintain the patient on complete bed rest.

The nurse is caring for a boy who has muscular dystrophy. When planning assistance with the patient's ADLs, what goal should the nurse prioritize? A) Promoting the patient's recovery from the disease B) Maximizing the patient's level of function C) Ensuring the patient's adherence to treatment D) Fostering the family's participation in care

B) Maximizing the patient's level of function

The nurse is caring for a patient who is postoperative following a craniotomy. When writing the plan of care, the nurse identifies a diagnosis of "deficient fluid volume related to fluid restriction and osmotic diuretic use." What would be an appropriate intervention for this diagnosis? A) Change the patient's position as indicated. B) Monitor serum electrolytes. C) Maintain NPO status. D) Monitor arterial blood gas (ABG) values.

B) Monitor serum electrolytes.

A patient with possible bacterial meningitis is admitted to the ICU. What assessment finding would the nurse expect for a patient with this diagnosis? A) Pain upon ankle dorsiflexion of the foot B) Neck flexion produces flexion of knees and hips C) Inability to stand with eyes closed and arms extended without swaying D) Numbness and tingling in the lower extremities

B) Neck flexion produces flexion of knees and hips

A patient with MS has developed dysphagia as a result of cranial nerve dysfunction. What nursing action should the nurse consequently perform? A) Arrange for the patient to receive a low residue diet. B) Position the patient upright during feeding. C) Suction the patient following each meal. D) Withhold liquids until the patient has finished eating.

B) Position the patient upright during feeding.

The nurse is caring for a patient recovering from an ischemic stroke. What intervention best addresses a potential complication after an ischemic stroke? A) Providing frequent small meals rather than three larger meals B) Teaching the patient to perform deep breathing and coughing exercises C) Keeping a urinary catheter in situ for the full duration of recovery D) Limiting intake of insoluble fiber

B) Teaching the patient to perform deep breathing and coughing exercises

A patient with metastatic cancer has developed trigeminal neuralgia and is taking carbamazepine (Tegretol) for pain relief. What principle applies to the administration of this medication? A) Tegretol is not known to have serious adverse effects. B) The patient should be monitored for bone marrow depression. C) Side effects of the medication include renal dysfunction. D) The medication should be first taken in the maximum dosage form to be effective.

B) The patient should be monitored for bone marrow depression.

A patient diagnosed with a pituitary adenoma has arrived on the neurologic unit. When planning the patient's care, the nurse should be aware that the effects of the tumor will primarily depend on what variable? A) Whether the tumor utilizes aerobic or anaerobic respiration B) The specific hormones secreted by the tumor C) The patient's pre-existing health status D) Whether the tumor is primary or the result of metastasis

B) The specific hormones secreted by the tumor

The nurse is performing stroke risk screenings at a hospital open house. The nurse has identified four patients who might be at risk for a stroke. Which patient is likely at the highest risk for a hemorrhagic stroke? A) White female, age 60, with history of excessive alcohol intake B) White male, age 60, with history of uncontrolled hypertension C) Black male, age 60, with history of diabetes D) Black male, age 50, with history of smoking

B) White male, age 60, with history of uncontrolled hypertension

A patient with Parkinson's disease is experiencing episodes of constipation that are becoming increasingly frequent and severe. The patient states that he has been achieving relief for the past few weeks by using OTC laxatives. How should the nurse respond? A) "It's important to drink plenty of fluids while you're taking laxatives." B) "Make sure that you supplement your laxatives with a nutritious diet." C) "Let's explore other options, because laxatives can have side effects and create dependency." D) "You should ideally be using herbal remedies rather than medications to promote bowel function."

C) "Let's explore other options, because laxatives can have side effects and create dependency."

The nurse is caring for a patient newly diagnosed with a primary brain tumor. The patient asks the nurse where his tumor came from. What would be the nurse's best response? A) "Your tumor originated from somewhere outside the CNS." B) "Your tumor likely started out in one of your glands." C) "Your tumor originated from cells within your brain itself." D) "Your tumor is from nerve tissue somewhere in your body."

C) "Your tumor originated from cells within your brain itself."

The pathophysiology of an ischemic stroke involves the ischemic cascade, which includes the following steps: 1. Change in pH 2. Blood flow decreases 3. A switch to anaerobic respiration 4. Membrane pumps fail 5. Cells cease to function 6. Lactic acid is generated Put these steps in order in which they occur. A) 635241 B) 352416 C) 236145 D) 162534

C) 236145

A neurologic nurse is reviewing seizures with a group of staff nurses. How should this nurse best describe the cause of a seizure? A) Sudden electrolyte changes throughout the brain B) A dysrhythmia in the peripheral nervous system C) A dysrhythmia in the nerve cells in one section of the brain D) Sudden disruptions in the blood flow throughout the brain

C) A dysrhythmia in the nerve cells in one section of the brain

A patient who has been on long-term phenytoin (Dilantin) therapy is admitted to the unit. In light of the adverse of effects of this medication, the nurse should prioritize which of the following in the patient's plan of care? A) Monitoring of pulse oximetry B) Administration of a low-protein diet C) Administration of thorough oral hygiene D) Fluid restriction as ordered

C) Administration of thorough oral hygiene

A community health nurse is giving an educational presentation about stroke and heart disease at the local senior citizens center. What nonmodifiable risk factor for stroke should the nurse cite? A) Female gender B) Asian American race C) Advanced age D) Smoking

C) Advanced age

The nurse is reviewing the medication administration record of a female patient who possesses numerous risk factors for stroke. Which of the woman's medications carries the greatest potential for reducing her risk of stroke? A) Naproxen 250 PO b.i.d. B) Calcium carbonate 1,000 mg PO b.i.d. C) Aspirin 81 mg PO o.d. D) Lorazepam 1 mg SL b.i.d. PRN

C) Aspirin 81 mg PO o.d.

A patient with a brain tumor has begun to exhibit signs of cachexia. What subsequent assessment should the nurse prioritize? A) Assessment of peripheral nervous function B) Assessment of cranial nerve function C) Assessment of nutritional status D) Assessment of respiratory status

C) Assessment of nutritional status

A patient with a new diagnosis of ischemic stroke is deemed to be a candidate for treatment with tissue plasminogen activator (t-PA) and has been admitted to the ICU. In addition to closely monitoring the patient's cardiac and neurologic status, the nurse monitors the patient for signs of what complication? A) Acute pain B) Septicemia C) Bleeding D) Seizures

C) Bleeding

A 33-year-old patient presents at the clinic with complaints of weakness, incoordination, dizziness, and loss of balance. The patient is hospitalized and diagnosed with MS. What sign or symptom, revealed during the initial assessment, is typical of MS? A) Diplopia, history of increased fatigue, and decreased or absent deep tendon reflexes B) Flexor spasm, clonus, and negative Babinski's reflex C) Blurred vision, intention tremor, and urinary hesitancy D) Hyperactive abdominal reflexes and history of unsteady gait and episodic paresthesia in both legs

C) Blurred vision, intention tremor, and urinary hesitancy

The nurse responds to the call light of a patient who has had a cervical diskectomy earlier in the day. The patient states that she is having severe pain that had a sudden onset. What is the nurse's most appropriate action? A) Palpate the surgical site. B) Remove the dressing to assess the surgical site. C) Call the surgeon to report the patient's pain. D) Administer a dose of an NSAID.

C) Call the surgeon to report the patient's pain.

A patient is admitted through the ED with suspected St. Louis encephalitis. The unique clinical feature of St. Louis encephalitis will make what nursing action a priority? A) Serial assessments of hemoglobin levels B) Blood glucose monitoring C) Close monitoring of fluid balance D) Assessment of pain along dermatomes

C) Close monitoring of fluid balance

The nurse is discharging home a patient who suffered a stroke. He has a flaccid right arm and leg and is experiencing problems with urinary incontinence. The nurse makes a referral to a home health nurse because of an awareness of what common patient response to a change in body image? A) Denial B) Fear C) Depression D) Disassociation

C) Depression

A patient is being admitted to the neurologic ICU with suspected herpes simplex virus encephalitis. What nursing action best addresses the patient's complaints of headache? A) Initiating a patient-controlled analgesia (PCA) of morphine sulfate B) Administering hydromorphone (Dilaudid) IV as needed C) Dimming the lights and reducing stimulation D) Distracting the patient with activity

C) Dimming the lights and reducing stimulation

A patient, brought to the clinic by his wife and son, is diagnosed with Huntington disease. When providing anticipatory guidance, the nurse should address the future possibility of what effect of Huntington disease? A) Metastasis B) Risk for stroke C) Emotional and personality changes D) Pathologic bone fractures

C) Emotional and personality changes

A patient has experienced a seizure in which she became rigid and then experienced alternating muscle relaxation and contraction. What type of seizure does the nurse recognize? A) Unclassified seizure B) Absence seizure C) Generalized seizure D) Focal seizure

C) Generalized seizure

A patient diagnosed with a hemorrhagic stroke has been admitted to the neurologic ICU. The nurse knows that teaching for the patient and family needs to begin as soon as the patient is settled on the unit and will continue until the patient is discharged. What will family education need to include? A) How to differentiate between hemorrhagic and ischemic stroke B) Risk factors for ischemic stroke C) How to correctly modify the home environment D) Techniques for adjusting the patient's medication dosages at home

C) How to correctly modify the home environment

A patient with amyotrophic lateral sclerosis (ALS) is being visited by the home health nurse who is creating a care plan. What nursing diagnosis is most likely for a patient with this condition? A) Chronic confusion B) Impaired urinary elimination C) Impaired verbal communication D) Bowel incontinence

C) Impaired verbal communication

A nurse is planning the care of a 28-year-old woman hospitalized with a diagnosis of myasthenia gravis. What approach would be most appropriate for the care and scheduling of diagnostic procedures for this patient? A) All at one time, to provide a longer rest period B) Before meals, to stimulate her appetite C) In the morning, with frequent rest periods D) Before bedtime, to promote rest

C) In the morning, with frequent rest periods

The nurse educator is discussing neoplasms with a group of recent graduates. The educator explains that the effects of neoplasms are caused by the compression and infiltration of normal tissue. The physiologic changes that result can cause what pathophysiologic events? Select all that apply. A) Intracranial hemorrhage B) Infection of cerebrospinal fluid C) Increased ICP D) Focal neurologic signs E) Altered pituitary function

C) Increased ICP D) Focal neurologic signs E) Altered pituitary function

A patient is being admitted to the neurologic ICU following an acute head injury that has resulted in cerebral edema. When planning this patient's care, the nurse would expect to administer what priority medication? A) Hydrochlorothiazide (HydroDIURIL) B) Furosemide (Lasix) C) Mannitol (Osmitrol) D) Spirolactone (Aldactone)

C) Mannitol (Osmitrol)

A patient with increased ICP has a ventriculostomy for monitoring ICP. The nurse's most recent assessment reveals that the patient is now exhibiting nuchal rigidity and photophobia. The nurse would be correct in suspecting the presence of what complication? A) Encephalitis B) CSF leak C) Meningitis D) Catheter occlusion

C) Meningitis

The critical care nurse is caring for 25-year-old man admitted to the ICU with a brain abscess. What is a priority nursing responsibility in the care of this patient? A) Maintaining the patient's functional independence B) Providing health education C) Monitoring neurologic status closely D) Promoting mobility

C) Monitoring neurologic status closely

A patient is recovering from intracranial surgery that was performed using the transsphenoidal approach. The nurse should be aware that the patient may have required surgery on what neurologic structure? A) Cerebellum B) Hypothalamus C) Pituitary gland D) Pineal gland

C) Pituitary gland

1. A patient has had an ischemic stroke and has been admitted to the medical unit. What action should the nurse perform to best prevent joint deformities? A) Place the patient in the prone position for 30 minutes/day. B) Assist the patient in acutely flexing the thigh to promote movement. C) Place a pillow in the axilla when there is limited external rotation. D) Place patient's hand in pronation.

C) Place a pillow in the axilla when there is limited external rotation.

A patient with Guillain-Barré syndrome has experienced a sharp decline in vital capacity. What is the nurse's most appropriate action? A) Administer bronchodilators as ordered. B) Remind the patient of the importance of deep breathing and coughing exercises. C) Prepare to assist with intubation. D) Administer supplementary oxygen by nasal cannula.

C) Prepare to assist with intubation.

The critical care nurse is admitting a patient in myasthenic crisis to the ICU. The nurse should prioritize what nursing action in the immediate care of this patient? A) Suctioning secretions B) Facilitating ABG analysis C) Providing ventilatory assistance D) Administering tube feedings

C) Providing ventilatory assistance

A patient with Parkinson's disease is undergoing a swallowing assessment because she has recently developed adventitious lung sounds. The patient's nutritional needs should be met by what method? A) Total parenteral nutrition (TPN) B) Provision of a low-residue diet C) Semisolid food with thick liquids D) Minced foods and a fluid restriction

C) Semisolid food with thick liquids

A nurse is admitting a patient with a severe migraine headache and a history of acute coronary syndrome. What migraine medication would the nurse question for this patient? A) Rizatriptan (Maxalt) B) Naratriptan (Amerge) C) Sumatriptan succinate (Imitrex) D) Zolmitriptan (Zomig)

C) Sumatriptan succinate (Imitrex)

Following a traumatic brain injury, a patient has been in a coma for several days. Which of the following statements is true of this patient's current LOC? A) The patient occasionally makes incomprehensible sounds. B) The patient's current LOC will likely become a permanent state. C) The patient may occasionally make nonpurposeful movements. D) The patient is incapable of spontaneous respirations.

C) The patient may occasionally make nonpurposeful movements.

A 48-year-old patient has been diagnosed with trigeminal neuralgia following recent episodes of unilateral face pain. The nurse should recognize what implication of this diagnosis? A) The patient will likely require lifelong treatment with anticholinergic medications. B) The patient has a disproportionate risk of developing myasthenia gravis later in life. C) The patient needs to be assessed for MS. D) The disease is self-limiting and the patient will achieve pain relief over time.

C) The patient needs to be assessed for MS.

The nurse is caring for a patient diagnosed with an ischemic stroke and knows that effective positioning of the patient is important. Which of the following should be integrated into the patient's plan of care? A) The patient's hip joint should be maintained in a flexed position. B) The patient should be in a supine position unless ambulating. C) The patient should be placed in a prone position for 15 to 30 minutes several times a day. D) The patient should be placed in a Trendelenberg position two to three times daily to promote cerebral perfusion.

C) The patient should be placed in a prone position for 15 to 30 minutes several times a day.

A nurse is collaborating with the interdisciplinary team to help manage a patient's recurrent headaches. What aspect of the patient's health history should the nurse identify as a potential contributor to the patient's headaches? A) The patient leads a sedentary lifestyle. B) The patient takes vitamin D and calcium supplements. C) The patient takes vasodilators for the treatment of angina. D) The patient has a pattern of weight loss followed by weight gain.

C) The patient takes vasodilators for the treatment of angina.

What should the nurse suspect when hourly assessment of urine output on a patient postcraniotomy exhibits a urine output from a catheter of 1,500 mL for two consecutive hours? A) Cushing syndrome B) Syndrome of inappropriate antidiuretic hormone (SIADH) C) Adrenal crisis D) Diabetes insipidus

D) Diabetes insipidus

A patient who was diagnosed with Parkinson's disease several months ago recently began treatment with levodopa-carbidopa. The patient and his family are excited that he has experienced significant symptom relief. The nurse should be aware of what implication of the patient's medication regimen? A) The patient is in a "honeymoon period" when adverse effects of levodopa-carbidopa are not yet evident. B) Benefits of levodopa-carbidopa do not peak until 6 to 9 months after the initiation of treatment. C) The patient's temporary improvement in status is likely unrelated to levodopa-carbidopa. D) Benefits of levodopa-carbidopa often diminish after 1 or 2 years of treatment.

D) Benefits of levodopa-carbidopa often diminish after 1 or 2 years of treatment.

A middle-aged woman has sought care from her primary care provider and undergone diagnostic testing that has resulted in a diagnosis of MS. What sign or symptom is most likely to have prompted the woman to seek care? A) Cognitive declines B) Personality changes C) Contractures D) Difficulty in coordination

D) Difficulty in coordination

A nursing student is writing a care plan for a newly admitted patient who has been diagnosed with a stroke. What major nursing diagnosis should most likely be included in the patient's plan of care? A) Adult failure to thrive B) Post-trauma syndrome C) Hyperthermia D) Disturbed sensory perception

D) Disturbed sensory perception

What should be included in the patient's care plan when establishing an exercise program for a patient affected by a stroke? A) Schedule passive range of motion every other day. B) Keep activity limited, as the patient may be over stimulated. C) Have the patient perform active range-of-motion (ROM) exercises once a day. D) Exercise the affected extremities passively four or five times a day.

D) Exercise the affected extremities passively four or five times a day.

The nurse is creating a plan of care for a patient who has a recent diagnosis of MS. Which of the following should the nurse include in the patient's care plan? A) Encourage patient to void every hour. B) Order a low-residue diet. C) Provide total assistance with all ADLs. D) Instruct the patient on daily muscle stretching.

D) Instruct the patient on daily muscle stretching.

During the examination of an unconscious patient, the nurse observes that the patient's pupils are fixed and dilated. What is the most plausible clinical significance of the nurse's finding? A) It suggests onset of metabolic problems. B) It indicates paralysis on the right side of the body. C) It indicates paralysis of cranial nerve X. D) It indicates an injury at the midbrain level.

D) It indicates an injury at the midbrain level.

A patient has a poor prognosis after being involved in a motor vehicle accident resulting in a head injury. As the patient's ICP increases and condition worsens, the nurse knows to assess for indications of approaching death. These indications include which of the following? A) Hemiplegia B) Dry mucous membranes C) Signs of internal bleeding D) Loss of brain stem reflexes

D) Loss of brain stem reflexes

A 37-year-old man is brought to the clinic by his wife because he is experiencing loss of motor function and sensation. The physician suspects the patient has a spinal cord tumor and hospitalizes him for diagnostic testing. In light of the need to diagnose spinal cord compression from a tumor, the nurse will most likely prepare the patient for what test? A) Anterior-posterior x-ray B) Ultrasound C) Lumbar puncture D) MRI

D) MRI

A 73-year-old man comes to the clinic complaining of weakness and loss of sensation in his feet and legs. Assessment of the patient shows decreased reflexes bilaterally. Why would it be a challenge to diagnose a peripheral neuropathy in this patient? A) Older adults are often vague historians. B) The elderly have fewer peripheral nerves than younger adults. C) Many older adults are hesitant to admit that their body is changing. D) Many symptoms can be the result of normal aging process.

D) Many symptoms can be the result of normal aging process.

The nurse is caring for a patient with increased intracranial pressure (ICP). The patient has a nursing diagnosis of "ineffective cerebral tissue perfusion." What would be an expected outcome that the nurse would document for this diagnosis? A) Copes with sensory deprivation. B) Registers normal body temperature. C) Pays attention to grooming. D) Obeys commands with appropriate motor responses.

D) Obeys commands with appropriate motor responses.

A patient presents at the clinic complaining of pain and weakness in her hands. On assessment, the nurse notes diminished reflexes in the upper extremities bilaterally and bilateral loss of sensation. The nurse knows that these findings are indicative of what? A) Guillain-Barré syndrome B) Myasthenia gravis C) Trigeminal neuralgia D) Peripheral nerve disorder

D) Peripheral nerve disorder

The nurse is caring for a patient who has undergone supratentorial removal of a pituitary mass. What medication would the nurse expect to administer prophylactically to prevent seizures in this patient? A) Prednisone B) Dexamethasone C) Cafergot D) Phenytoin

D) Phenytoin

A female patient is diagnosed with a right-sided stroke. The patient is now experiencing hemianopsia. How might the nurse help the patient manage her potential sensory and perceptional difficulties? A) Keep the lighting in the patient's room low. B) Place the patient's clock on the affected side. C) Approach the patient on the side where vision is impaired. D) Place the patient's extremities where she can see them.

D) Place the patient's extremities where she can see them.

After a subarachnoid hemorrhage, the patient's laboratory results indicate a serum sodium level of less than 126 mEq/L. What is the nurse's most appropriate action? A) Administer a bolus of normal saline as ordered. B) Prepare the patient for thrombolytic therapy as ordered. C) Facilitate testing for hypothalamic dysfunction. D) Prepare to administer 3% NaCl by IV as ordered.

D) Prepare to administer 3% NaCl by IV as ordered.

The nurse is providing care for a patient who is withdrawing from heavy alcohol use. The nurse and other members of the care team are present at the bedside when the patient has a seizure. In preparation for documenting this clinical event, the nurse should note which of the following? A) The ability of the patient to follow instructions during the seizure. B) The success or failure of the care team to physically restrain the patient. C) The patient's ability to explain his seizure during the postictal period. D) The patient's activities immediately prior to the seizure.

D) The patient's activities immediately prior to the seizure.

Neurological testing of the patient by the nurse indicates impaired functioning of the left Glossopharyngeal nerve (CN IX) and the Vagus nerve (CN X) Based on these findings the nurse plans to:

Withhold oral fluids or foods

11. You are a neuro trauma nurse working in a neuro ICU. What would you know is an acute emergency and is seen in clients with a cervical or high thoracic spinal cord injury after the spinal shock subsides?

a. Autonomic dysreflexia i. Autonomic dysreflexia is an acute emergency and is seen in clients with a cervical or high thoracic spinal cord injury, usually after the spinal shock subsidesautonomic dysreflexia develops in individuals with a neurologic level of spinal cord injury at or above the sixth thoracic vertebral level (T6). Autonomic dysreflexia causes an imbalanced reflex sympathetic discharge, leading to potentially life-threatening hypertension.

26. A patient in the emergency room has bruising over the mastoid bone and rhinorrhea. These are indicative of which type of skull fracture?

a. Basilar i. Bruising over the mastoid bone and rhinorrhea is indicative of a basilar skull fracture. A simple (linear) fracture is a break in the continuity of the bone. A comminuted fracture refers to a splintered or multiple fracture line.

30. At which level of cord injury does a patient have full head and neck control?

a. C5 i. At level C5, there is full head and neck control. At C1 there is little or no sensation or control of the head and neck. At C2 to C3 there is head and neck sensation and some neck control. At C4 there is good head and neck sensation and motor control

18. While snowboarding, a 17-year-old client fell and struck his head, resulting in a loss of consciousness. Upon his arrival via squad at the ED where you practice nursing, he regained consciousness within an hour. He was admitted for 24-hour observation and was discharged without neurologic impairment. What would you expect the neurologist's diagnosis to be?

a. Concussion i. A concussion results from a blow to the head that jars the brain. It usually is a consequence of falling, striking the head against a hard surface such as a windshield, colliding with another person (e.g., between athletes), battering during boxing, or being a victim of violence. The force of the blow causes temporary neurologic impairment but no serious damage to cerebral tissue. There is generally complete recovery within a short time.

22. A 24-year-old female rock climber is brought to the Emergency Department after a fall from the face of a rock. The young lady is admitted for observation after being diagnosed with a contusion to the brain. The client asks the nurse what having a contusion means. How should the nurse respond?

a. Contusions are bruising, and sometimes, hemorrhage of superficial cerebral tissue. i. Contusions result in bruising, and sometimes, hemorrhage of superficial cerebral tissue. When the head is struck directly, the injury to the brain is called a coup injury. Dual bruising can result if the force is strong enough to send the brain ricocheting to the opposite side of the skull, which is called a contrecoup injury. Edema develops at the site of or in areas opposite to the injury. A skull fracture can accompany a contusion

24. The nurse working on the neurological unit is caring for a client with a basilar skull fracture. During assessment, the nurse expects to observe Battle's sign, which is a sign of basilar skill fracture. Which of the following correctly describes Battle's sign?

a. Ecchymosis over the mastoid

31. The nursing instructor is teaching about hematomas to a pre-nursing pathophysiology class. What would the nursing instructor describe as an arterial bleed with rapid neurologic deterioration?

a. Epidural Hematoma i. An epidural hematoma stems from arterial bleeding, usually from the middle meningeal artery, and blood accumulation above the dura. It is characterized by rapidly progressive neurologic deterioration.

27. A client is admitted with a cervical spine injury sustained during a diving accident. When planning this client's care, the nurse should assign highest priority to which nursing diagnosis?

a. Ineffective breathing pattern

28. A patient with a concussion is discharged after the assessment. Which of the following instructions should the nurse give the patient's family?

a. Look for signs of increased intracranial pressure

7. A client with tetraplegia cannot do his own skin care. The nurse is teaching the caregiver about the importance of maintaining skin integrity. Which of the following will the nurse most encourage the caregiver to do?

a. Maintain a diet for the client that is high in protein, vitamins, and calories. i. To maintain healthy skin, the following interventions are necessary: regularly relieve pressure, protect from injury, keep clean and dry, avoid wrinkles in the bed, and maintain a diet high in protein, vitamins, and calories to ensure minimal wasting of muscles and healthy skin.

13. A 67-year-old client who was involved in a motor-vehicle collision is brought to the emergency department. After examination and diagnostics, the neurosurgeon diagnoses an epidural hematoma and orders that the client be prepared for surgery. In preparing the client for surgery, which of the following would be the least likely intervention you would expect?

a. Preoperative sedation

6. A client who has been severely beaten is admitted to the emergency department. The nurse suspects a basilar skull fracture after assessing:

a. Racoon's eyes and Bettle sign i. A basilar skull fracture commonly causes only periorbital ecchymosis (raccoon's eyes) and postmastoid ecchymosis (Battle sign); however, it sometimes also causes otorrhea, rhinorrhea, and loss of cranial nerve I (olfactory nerve) function.

10. A client who is disoriented and restless after sustaining a concussion during a car accident is admitted to the hospital. Which nursing diagnosis takes the highest priority in this client's care plan?

a. Risk for injury

20. A client admitted with a cerebral contusion is confused, disoriented, and restless. Which nursing diagnosis takes the highest priority?

a. Risk for injury related to neurologic deficit

14. A client has sustained a traumatic brain injury with involvement of the hypothalamus. The nurse is concerned about the development of diabetes insipidus. Which of the following would be an appropriate nursing intervention to monitor for early signs of diabetes insipidus?

a. Take daily weights. i. A record of daily weights is maintained for the client with a traumatic brain injury, especially if the client has hypothalamic involvement and is at risk for the development of diabetes insipidus. A weight loss will alert the nurse to possible fluid imbalance early in the process

1.A client in the intensive care unit (ICU) has a traumatic brain injury. The nurse must implement interventions to help control intracranial pressure (ICP). Which of the following are appropriate interventions to help control ICP?

a.Keep the client's neck in a neutral position (no flexing).


Ensembles d'études connexes

CH'S 6-11 terms to know for Test 2

View Set

Parasitology Quiz Questions (Part I)

View Set

Civil War and Reconstruction APUSH

View Set

COMM 131: Chapter 18 (Commemorative Speech)

View Set

Help Desk Final (Chapter 1 - 11) ALL

View Set